Violympic toán 9

Bạn chưa đăng nhập. Vui lòng đăng nhập để hỏi bài
Clgt

Cho a,b,c > 0 thoả mãn ab + bc +ca\(\ge\)3 . Tìm giá trị nhỏ nhất của: \(P=\frac{a^3}{1+b}+\frac{b^3}{1+c}+\frac{c^3}{1+a}\)

Nguyễn Việt Lâm
8 tháng 3 2020 lúc 15:48

\(\frac{a^3}{1+b}+\frac{1+b}{4}+\frac{1}{2}\ge\frac{3}{2}a\) ; \(\frac{b^3}{1+c}+\frac{1+c}{4}+\frac{1}{2}\ge\frac{3}{2}b\) ; \(\frac{c^3}{1+a}+\frac{1+a}{4}+\frac{1}{2}\ge\frac{3}{2}c\)

Cộng vế với vế:

\(P+\frac{1}{4}\left(a+b+c\right)+\frac{9}{4}\ge\frac{3}{2}\left(a+b+c\right)\)

\(\Rightarrow P\ge\frac{5}{4}\left(a+b+c\right)-\frac{9}{4}\ge\frac{5}{4}\sqrt{3\left(ab+bc+ca\right)}-\frac{9}{4}\ge\frac{5}{4}\sqrt{9}-\frac{9}{4}=\frac{3}{2}\)

Dấu "=" xảy ra khi \(a=b=c=1\)

Khách vãng lai đã xóa

Các câu hỏi tương tự
Luân Đào
Xem chi tiết
khoimzx
Xem chi tiết
asssssssaasawdd
Xem chi tiết
khoimzx
Xem chi tiết
linh angela nguyễn
Xem chi tiết
EDOGAWA CONAN
Xem chi tiết
Văn Thắng Hồ
Xem chi tiết
dbrby
Xem chi tiết
bach nhac lam
Xem chi tiết